You are on page 1of 128

1. RCGP curriculum 3.

06 - Womens Health External links NICE 2008 Antenatal care guidelines A 21-year-old female presents for review. She is 14 weeks pregnant and has been seen by the midwives for her booking visit. There have been no pregnancy related problems to date. Tests taken revealed the following: Blood group: A Rhesus negative What is the most appropriate management regarding her rhesus status? A. Give first dose of anti-D at 28 weeks B. No action required unless antenatal vaginal blood loss C. Give first dose of anti-D as soon as possible D. Give anti-D just prior to delivery E. No action required

Rhesus negative woman - anti-D at 28 + 34 weeks NICE recommend giving rhesus negative woman anti-D at 28 weeks followed by a second dose at 34 weeks

Antenatal care: timetable NICE issued guidelines on routine care for the healthy pregnant woman in March 2008. They recommend: 1. 10 antenatal visits in the first pregnancy if uncomplicated 2. 7 antenatal visits in subsequent pregnancies if uncomplicated 3. women do not need to be seen by a consultant if the pregnancy is uncomplicated

Gestation 8 - 12 weeks (ideally < 10 weeks)

Purpose of visit Booking visit 2. general information e.g. diet, alcohol, smoking, folic acid, vitamin D, antenatal classes 3. BP, urine dipstick, check BMI Booking bloods/urine FBC, blood group, rhesus status, red cell alloantibodies, haemoglobinopathies hepatitis B, syphilis, rubella HIV test is offered to all women urine culture to detect asymptomatic bacteriuria

10 - 13 weeks

Early scan to confirm dates, exclude multiple pregnancy

11 - 13+6 weeks 16 weeks

Down's syndrome screening including nuchal scan Information on the anomaly and the blood results. If Hb < 11 g/dl consider iron Routine care: BP and urine dipstick Anomaly scan Routine care: BP, urine dipstick, symphysis-fundal height (SFH) Routine care: BP, urine dipstick, SFH Second screen for anaemia and atypical red cell alloantibodies. If Hb < 10.5 g/dl consider iron First dose of anti-D prophylaxis to rhesus negative women Routine care as above Routine care as above Second dose of anti-D prophylaxis to rhesus negative women Information on labour and birth plan Routine care as above Check presentation - offer external cephalic version if indicated Information on breast feeding, vitamin K, 'babyblues' Routine care as above Routine care as above

18 - 20+6 weeks 25 weeks (only if primip) 28 weeks

31 weeks (only if primip) 34 weeks

36 weeks

38 weeks 40 weeks (only if

primip) 41 weeks

Discussion about options for prolonged pregnancy Routine care as above Discuss labour plans and possibility of induction

1- External links Australian goverment Prescribing in pregnancy A 26-year-old woman with a history of hypothyroidism and antiphospholipid syndrome becomes pregnant. Which one of the following is contraindicated in pregnancy? A. Aspirin B. Low-molecular weight heparin C. Warfarin D. Levothyroxine E. Unfractionated heparin Warfarin is contraindicated in pregnancy. Most women are switched to low-molecular weight heparin for the duration of the pregnancy. Prescribing in pregnant patients Very few drugs are known to be completely safe in pregnancy. The list below largely comprises of those known to be harmful.

Some countries have developed a grading system - see the link. Antibiotics 1. 2. 3. 4. tetracyclines aminoglycosides sulphonamides and trimethoprim quinolones: the BNF advises to avoid due to arthropathy in some animal studies

Other drugs 1. 2. 3. 4. 5. 6. ACE inhibitors, angiotensin II receptor antagonists statins warfarin sulfonylureas retinoids (including topical) cytotoxic agents

The majority of antiepileptics including valproate, carbamazepine and phenytoin are known to be potentially harmful. The decision to stop such treatments however is difficult as uncontrolled epilepsy is also a risk 2- RCGP curriculum 3.06 - Womens Health External links NICE 2008 Antenatal care guidelines Theme: Routine antenatal care

A. 7 - 8 weeks B. 8 - 12 weeks C. 11 - 13+6 weeks D. 14 -15 weeks E. 16 weeks F. 18 - 20+6 weeks G. 25 weeks H. 28 weeks I. 34 weeks J. 38 weeks For each of the following components of routine antenatal care select the gestation when it should occur 2. Anomaly scan 18 - 20+6 weeks 3. Down's syndrome screening where nuchal scanning is available 11 - 13+6 weeks

4.

Booking visit 8 - 12 weeks

Antenatal care: timetable

NICE issued guidelines on routine care for the healthy pregnant woman in March 2008. They recommend: 1. 2. 3. 10 antenatal visits in the first pregnancy if uncomplicated 7 antenatal visits in subsequent pregnancies if uncomplicated women do not need to be seen by a consultant if the pregnancy is uncomplicated

Gestation 8 - 12 weeks (ideally < 10 weeks)

Purpose of visit Booking visit 1. general information e.g. diet, alcohol, smoking, folic acid, vitamin D, antenatal classes BP, urine dipstick, check BMI

2.

Booking bloods/urine 1. 2. 3. FBC, blood group, rhesus status, red cell alloantibodies, haemoglobinopathies hepatitis B, syphilis, rubella HIV test is offered to all women

4.

urine culture to detect asymptomatic bacteriuria

10 - 13 weeks

Early scan to confirm dates, exclude multiple pregnancy Down's syndrome screening including nuchal scan Information on the anomaly and the blood results. If Hb < 11 g/dl consider iron Routine care: BP and urine dipstick Anomaly scan Routine care: BP, urine dipstick, symphysis-fundal height (SFH) Routine care: BP, urine dipstick, SFH Second screen for anaemia and atypical red cell alloantibodies. If Hb < 10.5 g/dl consider iron First dose of anti-D prophylaxis to rhesus negative women Routine care as above

11 - 13+6 weeks 16 weeks

18 - 20+6 weeks 25 weeks (only if primip) 28 weeks

31 weeks (only if primip) 34 weeks

Routine care as above Second dose of anti-D prophylaxis to rhesus

negative women Information on labour and birth plan 36 weeks Routine care as above Check presentation - offer external cephalic version if indicated Information on breast feeding, vitamin K, 'babyblues' Routine care as above Routine care as above Discussion about options for prolonged pregnancy Routine care as above Discuss labour plans and possibility of induction

38 weeks 40 weeks (only if primip) 41 weeks

3- RCGP curriculum 3.06 - Womens Health Theme: Bleeding in pregnancy A. Placental abruption B. Ectopic pregnancy C. Threatened miscarriage D. Hydatidiform mole

E. Vasa praevia F. Placenta praevia G. Bloody show H. Missed (delayed) miscarriage I. Inevitable miscarriage For each one of the following scenarios select the most likely diagnosis: 1. A 25-year-old woman at 25 weeks gestation presents with constant lower abdominal pain and a small amount of vaginal bleeding. On examination blood pressure is 90 / 60 mmHg Placental abruption 2. A 31-year-old woman presents with painless vaginal bleeding at 15 weeks gestation. She has not yet had any antenatal care despite suffering from severe vomiting. On examination the uterus is large for dates Hydatidiform mole 3. A 19-year-old woman presents with a two day history of central lower abdominal pain and one day history of vaginal bleeding. Her last period was 8 weeks ago. On examination her cervix is tender to touch

Ectopic pregnancy Bleeding in pregnancy

The table below outlines the major causes of bleeding during pregnancy. Antepartum haemorrhage is defined as bleeding after 24 weeks 1st trimester Spontaneous abortion Ectopic pregnancy Hydatidiform mole 2nd trimester Spontaneous abortion Hydatidiform mole Placental abruption 3rd trimester Bloody show Placental abruption Placenta praevia Vasa praevia

Alongside the pregnancy related causes, conditions such as sexually transmitted infections and cervical polyps should be excluded. The table below outlines the key features of each condition: Spontaneous abortion Threatened miscarriage - painless vaginal bleeding typically around 6-9 weeks Missed (delayed) miscarriage - light vaginal bleeding and symptoms of pregnancy disappear Inevitable miscarriage - complete or incomplete depending or whether all fetal and placental tissue has been expelled. Incomplete miscarriage - heavy bleeding and crampy, lower abdo pain. Complete miscarriage - little bleeding

Ectopic pregnancy

Typically history of 6-8 weeks amenorrhoea with lower abdominal pain (usually unilateral) initially and vaginal bleeding later. Shoulder tip pain and cervical excitation may be present Typically bleeding in first or early second trimester associated with exaggerated symptoms of pregnancy e.g. hyperemesis. The uterus may be large for dates and serum hCG is very high Constant lower abdominal pain and, woman may be more shocked than is expected by visible blood loss. Tender, tense uterus* with normal lie and presentation. Fetal heart may be distressed Vaginal bleeding, no pain. Non-tender uterus* but lie and presentation may be abnormal Rupture of membranes followed immediately by vaginal bleeding. Fetal bradycardia is classically seen

Hydatidiform mole

Placental abruption

Placental praevia Vasa praevia

*vaginal examination should not be performed in primary care for suspected antepartum haemorrhage - women with placenta praevia may haemorrhage 4- RCGP curriculum 3.06 - Womens Health External links

NICE 2008 Management of diabetes in pregnancy SIGN Management of diabetes in pregnancy Postgraduate Medical Journal DKA in pregnancy You review a 28-year-old woman who is 26 weeks pregnant. She has just had a routine glucose tolerance test as her BMI is 34 kg/m^2. The following results were obtained: Time (hours) Blood glucose (mmol/l) 0 2 7.1 11.2

There have been no other antenatal problems and her anomaly scan was normal. What is the most appropriate action? A. Start metformin + advice about diet / exercise B. Start metformin + advice about diet / exercise + self-monitor glucose levels C. Advice about diet / exercise + self-monitor glucose levels

D. Start insulin + advice about diet / exercise + selfmonitor glucose levels E. Advise weight loss + start metformin Most women (around 80%) with gestational diabetes can be managed with a combination of diet and self-monitoring. Pregnancy: diabetes mellitus Diabetes mellitus may be a pre-existing problem or develop during pregnancy, gestational diabetes. It complicates around 1 in 40 pregnancies Risk factors for gestational diabetes 1. 2. 3. 4. 5. BMI of > 30 kg/m^2 previous macrosomic baby weighing 4.5 kg or above. previous gestational diabetes first-degree relative with diabetes family origin with a high prevalence of diabetes (South Asian, black Caribbean and Middle Eastern)

Screening for gestational diabetes 1. if a women has had gestational diabetes previously an oral glucose tolerance test (OGTT) should be performed at 16-18 weeks and at 28 weeks if the first test is normal women with any of the other risk factors should be offered an OGTT at 24-28 weeks currently the same WHO diagnostic criteria are used as for non-pregnant patients. There is however increasing evidence that a lower threshold should be used as treating borderline patients improves both maternal and neonatal outcomes

2. 3.

NICE issued guidelines on the management of diabetes mellitus in pregnancy in 2008 Management of pre-existing diabetes 1. 2. 3. 4. 5. 6. weight loss for women with BMI of > 27 kg/m^2 stop oral hypoglycaemic agents, apart from metformin, and commence insulin folic acid 5 mg/day from pre-conception to 12 weeks gestation detailed anomaly scan at 18-20 weeks including fourchamber view of the heart and outflow tracts tight glycaemic control reduces complication rates treat retinopathy as can worsen during pregnancy

Management of gestational diabetes 1. 2. responds to changes in diet and exercise in around 80% of women oral hypoglycaemic agents (metformin or glibenclamide) or insulin injections are needed if blood glucose control is poor or this is any evidence of complications (e.g. macrosomia) there is increasing evidence that oral hypoglycaemic agents are both safe and give similar outcomes to insulin hypoglycaemic medication should be stopped following delivery a fasting glucose should be checked at the 6 week postnatal check 5- RCGP curriculum 3.06 - Womens Health External links NICE 2008 Antenatal care guidelines

3. 4. 5.

Which one of the following statements regarding routine antenatal care is incorrect? A. NICE recommend 10 antenatal visits in the first pregnancy if uncomplicated B. Women do not need to be seen by a consultant if the pregnancy is uncomplicated C. All women are offered a HIV test D. The early ultrasound scan and nuchal scan should not be done at the same time E. Women are screened twice during pregnancy for anaemia Many centres now perform the early ultrasound scan and nuchal scan at the same time Antenatal care: timetable NICE issued guidelines on routine care for the healthy pregnant woman in March 2008. They recommend: 1. 2. 3. 10 antenatal visits in the first pregnancy if uncomplicated 7 antenatal visits in subsequent pregnancies if uncomplicated women do not need to be seen by a consultant if the pregnancy is uncomplicated Purpose of visit

Gestation

8 - 12 weeks (ideally < 10 weeks)

Booking visit 1. general information e.g. diet, alcohol, smoking, folic acid, vitamin D, antenatal classes BP, urine dipstick, check BMI

2.

Booking bloods/urine 1. 2. 3. 4. FBC, blood group, rhesus status, red cell alloantibodies, haemoglobinopathies hepatitis B, syphilis, rubella HIV test is offered to all women urine culture to detect asymptomatic bacteriuria

10 - 13 weeks

Early scan to confirm dates, exclude multiple pregnancy Down's syndrome screening including nuchal scan Information on the anomaly and the blood results. If Hb < 11 g/dl consider iron Routine care: BP and urine dipstick Anomaly scan Routine care: BP, urine dipstick, symphysis-fundal height (SFH)

11 - 13+6 weeks 16 weeks

18 - 20+6 weeks 25 weeks (only if primip)

28 weeks

Routine care: BP, urine dipstick, SFH Second screen for anaemia and atypical red cell alloantibodies. If Hb < 10.5 g/dl consider iron First dose of anti-D prophylaxis to rhesus negative women Routine care as above

31 weeks (only if primip) 34 weeks

Routine care as above Second dose of anti-D prophylaxis to rhesus negative women Information on labour and birth plan Routine care as above Check presentation - offer external cephalic version if indicated Information on breast feeding, vitamin K, 'babyblues' Routine care as above Routine care as above Discussion about options for prolonged pregnancy Routine care as above Discuss labour plans and possibility of induction

36 weeks

38 weeks 40 weeks (only if primip) 41 weeks

6- External links NICE 2010 Hypertension in pregnancy A 36-year-old woman presents for a routine antenatal review. She is now 15 weeks pregnant. Her blood pressure in clinic is 154/94 mmHg. This is confirmed with ambulatory blood pressure monitoring. On reviewing the notes it appears her blood pressure four weeks ago was 146/88 mmHg. A urine dipstick shows is normal. There is no significant past medical history of note. What is the most likely diagnosis? A. Pre-eclampsia B. Pregnancy-induced hypertension C. White-coat hypertension D. Normal physiological change E. Pre-existing hypertension This lady has pre-existing hypertension. Pregnancy related blood pressure problems (such as pregnancy-induced hypertension or pre-eclampsia) do not occur before 20 weeks. The raised ambulatory blood pressure readings exclude a diagnosis of whitecoat hypertension. Note the use of the term 'pre-existing hypertension' rather than essential hypertension. Raised blood pressure in a 36-year-old female is not that common and raises the possibility of secondary hypertension. Hypertension in pregnancy

The classification of hypertension in pregnancy is complicated and varies. Remember, in normal pregnancy: 1. 2. blood pressure usually falls in the first trimester (particularly the diastolic), and continues to fall until 20-24 weeks after this time the blood pressure usually increases to prepregnancy levels by term

Hypertension in pregnancy in usually defined as: 1. 2. systolic > 140 mmHg or diastolic > 90 mmHg or an increase above booking readings of > 30 mmHg systolic or > 15 mmHg diastolic

After establishing that the patient is hypertensive they should be categorised into one of the following groups Pre-existing hypertension Pregnancy-induced hypertension (PIH, also known as gestational hypertension) Hypertension (as defined above) occurring in the second half of pregnancy (i.e. after 20 weeks) No proteinuria, no oedema Occurs in around 5-7% of pregnancies Pre-eclampsia

A history of hypertension before pregnancy or an elevated blood pressure > 140/90 mmHg before 20 weeks gestation No proteinuria, no oedema

Pregnancy-induced hypertension in association with proteinuria (> 0.3g / 24 hours) Oedema may occur but is now less commonly used as a criteria

Occurs in 3-5% of pregnancies and is more common in older women

Resolves following birth (typically after one month). Women with PIH are at increased risk of future pre-eclampsia or hypertension later in life

Occurs in around 5% of pregnancies

7- Question stats A 29-year-old pregnant woman is diagnosed as having gestational diabetes at 22 weeks gestation. Which one of the following complications is she at an increased risk of developing? A. Polyhydramnios B. Antepartum haemorrhage C. Pregnancy-induced hypertension D. Neonatal hyperglycaemia E. Postnatal depresion Pregnancy: Diabetes Complications: Maternal complications 1. 2. polyhydramnios - 25%, possibly due to fetal polyuria preterm labour - 15%, associated with polyhydramnios

Neonatal complications

1. 2. 3. 4. 5. 6. 7. 8. 9.

macrosomia (although diabetes may also cause small for gestational age babies) hypoglycaemia respiratory distress syndrome: surfactant production is delayed polycythaemia: therefore more neonatal jaundice malformation rates increase 3-4 fold e.g. sacral agenesis, CNS and CVS malformations (hypertrophic cardiomyopathy) stillbirth hypomagnesaemia hypocalcaemia shoulder dystocia (may cause Erb's palsy)

8- RCGP curriculum 3.06 - Womens Health External links NICE 2008 Antenatal care guidelines Theme: Routine antenatal care A. 7 - 8 weeks B. 8 - 12 weeks C. 10 - 13 weeks D. 14 -15 weeks E. 16 weeks

F. 18 - 20+6 weeks G. 25 weeks H. 28 weeks I. 34 weeks J. 38 weeks For each of the following components of routine antenatal care select the gestation when it should occur 4. Early scan to confirm dates 10 - 13 weeks 5. First screen for anaemia and atypical red cell alloantibodies 8 - 12 weeks 6. First dose of anti-D prophylaxis to rhesus negative women 28 weeks Antenatal care: timetable

NICE issued guidelines on routine care for the healthy pregnant woman in March 2008. They recommend:

1. 2. 3.

10 antenatal visits in the first pregnancy if uncomplicated 7 antenatal visits in subsequent pregnancies if uncomplicated women do not need to be seen by a consultant if the pregnancy is uncomplicated

Gestation 8 - 12 weeks (ideally < 10 weeks)

Purpose of visit Booking visit 1. general information e.g. diet, alcohol, smoking, folic acid, vitamin D, antenatal classes BP, urine dipstick, check BMI

2.

Booking bloods/urine 1. 2. 3. 4. FBC, blood group, rhesus status, red cell alloantibodies, haemoglobinopathies hepatitis B, syphilis, rubella HIV test is offered to all women urine culture to detect asymptomatic bacteriuria

10 - 13 weeks

Early scan to confirm dates, exclude multiple pregnancy Down's syndrome screening including nuchal scan Information on the anomaly and the blood

11 - 13+6 weeks 16 weeks

results. If Hb < 11 g/dl consider iron Routine care: BP and urine dipstick 18 - 20+6 weeks 25 weeks (only if primip) 28 weeks Anomaly scan Routine care: BP, urine dipstick, symphysis-fundal height (SFH) Routine care: BP, urine dipstick, SFH Second screen for anaemia and atypical red cell alloantibodies. If Hb < 10.5 g/dl consider iron First dose of anti-D prophylaxis to rhesus negative women Routine care as above

31 weeks (only if primip) 34 weeks

Routine care as above Second dose of anti-D prophylaxis to rhesus negative women Information on labour and birth plan Routine care as above Check presentation - offer external cephalic version if indicated Information on breast feeding, vitamin K, 'babyblues'

36 weeks

38 weeks 40 weeks (only if primip) 41 weeks

Routine care as above Routine care as above Discussion about options for prolonged pregnancy Routine care as above Discuss labour plans and possibility of induction

9- RCGP curriculum 3.06 - Womens Health External links NICE 2008 Management of diabetes in pregnancy SIGN Management of diabetes in pregnancy Postgraduate Medical Journal DKA in pregnancy A 25-year-old Asian woman who is 26 weeks pregnant has an oral glucose tolerance test (OGTT). This was requested due to a combination of her ethnicity and a background of obesity. A recent ultrasound shows that the fetus is large for dates. The following results are obtained:

Time (hours) Blood glucose (mmol/l) 0 2 9.2 14.2

What is the most appropriate management? A. Start insulin B. Give advice about a diabetic diet C. Give advice about a diabetic diet + repeat OGTT in 4 weeks D. Start gliclazide E. Start insulin + aspirin Insulin should be started straight away given the blood glucose levels and evidence of macrosomia. Some endocrinologists would consider using either metformin or glibenclamide but gliclazide (option D) is not mentioned in the NICE guidelines. Pregnancy: diabetes mellitus Diabetes mellitus may be a pre-existing problem or develop during pregnancy, gestational diabetes. It complicates around 1 in 40 pregnancies Risk factors for gestational diabetes 1. 2. BMI of > 30 kg/m^2 previous macrosomic baby weighing 4.5 kg or above.

3. 4. 5.

previous gestational diabetes first-degree relative with diabetes family origin with a high prevalence of diabetes (South Asian, black Caribbean and Middle Eastern)

Screening for gestational diabetes 1. if a women has had gestational diabetes previously an oral glucose tolerance test (OGTT) should be performed at 16-18 weeks and at 28 weeks if the first test is normal women with any of the other risk factors should be offered an OGTT at 24-28 weeks currently the same WHO diagnostic criteria are used as for non-pregnant patients. There is however increasing evidence that a lower threshold should be used as treating borderline patients improves both maternal and neonatal outcomes

2. 3.

NICE issued guidelines on the management of diabetes mellitus in pregnancy in 2008 Management of pre-existing diabetes 1. 2. 3. 4. 5. 6. weight loss for women with BMI of > 27 kg/m^2 stop oral hypoglycaemic agents, apart from metformin, and commence insulin folic acid 5 mg/day from pre-conception to 12 weeks gestation detailed anomaly scan at 18-20 weeks including fourchamber view of the heart and outflow tracts tight glycaemic control reduces complication rates treat retinopathy as can worsen during pregnancy

Management of gestational diabetes 1. responds to changes in diet and exercise in around 80% of women

2.

3. 4. 5.

oral hypoglycaemic agents (metformin or glibenclamide) or insulin injections are needed if blood glucose control is poor or this is any evidence of complications (e.g. macrosomia) there is increasing evidence that oral hypoglycaemic agents are both safe and give similar outcomes to insulin hypoglycaemic medication should be stopped following delivery a fasting glucose should be checked at the 6 week postnatal check

10-

RCGP curriculum

3.06 - Womens Health External links NICE 2008 Antenatal care guidelines A 23-year-old female primip presents during the 10th week of pregnancy complaining of persistent nausea. Urine dipstick shows no evidence of ketones. She request medication to help relieve the nausea. What is the most appropriate drug to prescribe? A. Ondansetron B. Metoclopramide C. Domperidone D. Aprepitant

E. Promethazine

Vomiting in pregnancy: antihistamines are first-line

Antenatal care: specific points NICE issued guidelines on routine care for the healthy pregnant woman in March 2008 Nausea and vomiting 1. 2. natural remedies - ginger and acupuncture on the 'p6' point (by the wrist) are recommended by NICE antihistamines should be used first-line (BNF suggests promethazine as first-line)

Vitamin D 1. NICE recommend 'All women should be informed at the booking appointment about the importance for their own and their baby's health of maintaining adequate vitamin D stores during pregnancy and whilst breastfeeding' 'women may choose to take 10 micrograms of vitamin D per day, as found in the Healthy Start multivitamin supplement' particular care should be taken with women at risk (e.g. Asian, obese, poor diet)

2. 3.

Alcohol 1. NICE recommend women should avoid alcohol during the first trimester

2.

if women choose to drink alcohol during pregnancy they should be advised to drink no more than 1 to 2 UK units once or twice a week 11RCGP curriculum

3.06 - Womens Health A 28-year-old woman 27 weeks into her first pregnancy presents with vaginal bleeding. Which one of the following features would point towards a diagnosis of placenta praevia rather than placenta abruption? A. Tender, tense uterus B. Normal lie and presentation C. No pain D. Distressed fetal heart E. Shock out of keeping with visible blood loss Antepartum haemorrhage: determining cause Antepartum haemorrhage is defined as bleeding from the genital tract after 24 weeks pregnancy, prior to delivery of the fetus Distinguishing placental abruption from praevia: Placental abruption: 1. 2. 3. 4. 5. shock out of keeping with visible loss pain constant tender, tense uterus* normal lie and presentation fetal heart: absent/distressed

6. 7.

coagulation problems beware pre-eclampsia, DIC, anuria Placenta Praevia

1. 2. 3. 4. 5. 6. 7.

Shock in proportion to visible loss No pain Uterus not tender* Lie and presentation may be abnormal Fetal heart usually normal Coagulation problems rare Small bleeds before large

*vaginal examination should not be performed in primary care for suspected antepartum haemorrhage - women with placenta praevia may haemorrhage 12RCGP curriculum

3.06 - Womens Health External links RCOG 2006 Management of severe pre-eclampsia/eclampsia NICE 2010 Hypertension in pregnancy You admit a woman who is 34 weeks pregnant to the obstetric ward. She has been monitored for the past few weeks due to pregnancy-induced hypertension but has now developed proteinuria. Her blood pressure is 162/94 mmHg. Which one of the following antihypertensives is she most likely to be commenced on?

A. Moxonidine B. Atenolol C. Labetalol D. Nidedipine E. Methyldopa

Labetalol is first-line for pregnancy-induced hypertension Pre-eclampsia Pre-eclampsia is a condition seen after 20 weeks gestation characterised by pregnancy-induced hypertension in association with proteinuria (> 0.3g / 24 hours). Oedema used to be third element of the classic triad but is now often not included in the definition as it is not specific Pre-eclampsia is important as it predisposes to the following problems 1. 2. 3. 4. 5. Fetal: prematurity, intrauterine growth retardation Eclampsia Haemorrhage: placental abruption, intra-abdominal, intracerebral Cardiac failure Multi-organ failure

Risk Factors

1. 2. 3. 4. 5. 6. 7. 8. 9.

> 40 years old Nulliparity (or new partner) Multiple pregnancy Body mass index > 30 kg/m^2 Diabetes mellitus Pregnancy interval of more than 10 years Family history of pre-eclampsia Previous history of pre-eclampsia Pre-existing vascular disease such as hypertension or renal disease

Features of severe pre-eclampsia 1. 2. 3. 4. 5. 6. 7. 8. Hypertension: typically > 170/110 mmHg and proteinuria as above Proteinuria: dipstick ++/+++ Headache Visual disturbance Papilloedema RUQ/Epigastric pain Hyperreflexia Platelet count < 100 * 106/l, abnormal liver enzymes or HELLP syndrome

Management 1. Consensus guidelines recommend treating blood pressure > 160/110 mmHg although many clinicians have a lower threshold Oral labetalol is now first-line following the 2010 NICE guidelines. Nifedipine and hydralazine may also be used Delivery of the baby is the most important and definitive management step. The timing depends on the individual clinical scenario

2. 3.

13-

RCGP curriculum

3.06 - Womens Health External links Australian goverment Prescribing in pregnancy A woman who is 24-weeks pregnant presents with a productive cough. On examination crackles can be heard in the left base and a decision is made to give an antibiotic. Which one of the following is least suitable to prescribe? A. Ciprofloxacin B. Erythromycin C. Co-amoxiclav D. Cefalexin E. Cefaclor The BNF advises avoiding quinolones in pregnancy due to arthropathy in animal studies Prescribing in pregnant patients Very few drugs are known to be completely safe in pregnancy. The list below largely comprises of those known to be harmful. Some countries have developed a grading system - see the link. Antibiotics to avoid:

1. 2. 3. 4.

Tetracyclines Aminoglycosides Sulphonamides and trimethoprim Quinolones: the BNF advises to avoid due to arthropathy in some animal studies

Other drugs 1. 2. 3. 4. 5. 6. ACE inhibitors, angiotensin II receptor antagonists Statins Warfarin Sulfonylureas Retinoids (including topical) Cytotoxic agents

The majority of antiepileptics including valproate, carbamazepine and phenytoin are known to be potentially harmful. The decision to stop such treatments however is difficult as uncontrolled epilepsy is also a risk 14RCGP curriculum

3.06 - Womens Health Which one of the following statements regarding hyperemesis gravidarum is correct? A. Most common between 12 - 16 weeks gestation B. Affects around 4% of pregnant women C. Wernicke's encephalopathy is a recognised complication

D. Ondansetron is first-line in women after 12 weeks gestation E. Commonly associated with hypothyroidism Hyperemesis gravidarum Hyperemesis gravidarum describes excessive vomiting during pregnancy. It occurs in around 1% of pregnancies and is thought to be related to raised beta hCG levels. Hyperemesis gravidarum is most common between 8 and 12 weeks but may persist up to 20 weeks*. Associations 1. 2. 3. 4. 5. multiple pregnancies trophoblastic disease hyperthyroidism nulliparity obesity

Smoking is associated with a decreased incidence of hyperemesis Management 1. 2. antihistamines should be used first-line (BNF suggests promethazine as first-line) admission may be needed for IV hydration

Complications 1. 2. 3. 4. Wernicke's encephalopathy Mallory-Weiss tear central pontine myelinolysis acute tubular necrosis

5.

fetal: small for gestational age, pre-term birth

*and in very rare cases beyond 20 weeks 15RCGP curriculum

3.06 - Womens Health A 24-year-old woman presents 8 days after giving birth. She complains of a persistent pink vaginal discharge which is 'smelly'. On examination her pulse is 90 / min, temperature 38.2C and she has diffuse suprapubic tenderness. On vaginal examination the uterus feels generally tender. Examination of her breasts is unremarkable. Urine dipstick shows blood ++. What is the most appropriate management? A. Arrange urgent ultrasound to exclude retained products + send MSSU + take high vaginal swab B. Send MSSU + take high vaginal swab + start oral co-amoxiclav + metronidazole C. Arrange urgent ultrasound to exclude retained products + send MSSU + start oral co-amoxiclav D. Admit to hospital E. Send MSSU + take high vaginal swab + start oral co-amoxiclav This woman by definition has puerperal pyrexia, likely secondary to endometritis. She needs to be admitted for intravenous antibiotics.

Puerperal pyrexia Puerperal pyrexia may be defined as a temperature of > 38C in the first 14 days following delivery. Causes: 1. 2. 3. 4. 5. endometritis: most common cause urinary tract infection wound infections (perineal tears + caesarean section) mastitis venous thromboembolism

Management 1. if endometritis is suspected the patient should be referred to hospital for intravenous antibiotics (clindamycin and gentamicin until afebrile for greater than 24 hours) 16RCGP curriculum

3.06 - Womens Health External links NICE 2010 Hypertension in pregnancy A 24-year-old female who is 10 weeks in to her first pregnancy presents for review. Her blood pressure today is 126/82 mmHg. What normally happens to blood pressure during pregnancy? A. Falls in first half of pregnancy before rising to prepregnancy levels before term B. Systolic + diastolic rises by < 10 mmHg

C. Systolic + diastolic falls by < 10 mmHg D. Rise in first half of pregnancy before falling to prepregnancy levels before term E. Doesn't change Hypertension in pregnancy The classification of hypertension in pregnancy is complicated and varies. Remember, in normal pregnancy: 1. 2. blood pressure usually falls in the first trimester (particularly the diastolic), and continues to fall until 20-24 weeks after this time the blood pressure usually increases to prepregnancy levels by term

Hypertension in pregnancy in usually defined as: 1. 2. Systolic > 140 mmHg or diastolic > 90 mmHg Or an increase above booking readings of > 30 mmHg systolic or > 15 mmHg diastolic

After establishing that the patient is hypertensive they should be categorized into one of the following groups Pre-existing hypertension Pregnancy-induced hypertension (PIH, also known as gestational hypertension) Hypertension (as Pre-eclampsia

A history of

Pregnancy-induced

hypertension before pregnancy or an elevated blood pressure > 140/90 mmHg before 20 weeks gestation No proteinuria, no oedema Occurs in 3-5% of pregnancies and is more common in older women

defined above) occurring in the second half of pregnancy (i.e. after 20 weeks) No proteinuria, no oedema Occurs in around 5-7% of pregnancies Resolves following birth (typically after one month). Women with PIH are at increased risk of future pre-eclampsia or hypertension later in life

hypertension in association with proteinuria (> 0.3g / 24 hours) Oedema may occur but is now less commonly used as a criteria Occurs in around 5% of pregnancies

17-

RCGP curriculum

A pregnant woman comes to see you for advice. The Down's syndrome screening tests have indicated she is at high risk and the midwives have discussed the option of an amniocentesis. Which one of the following best describes the timing of an amniocentesis? A. Performed at 13-14 weeks, karyotype available after 3 weeks B. Performed at 16 weeks, karyotype available after 3

days C. Performed at 14-15 weeks, karyotype available after 3 weeks D. Performed at 16 weeks, karyotype available after 3 weeks E. Performed at 14-15 weeks, karyotype available after 3 days

Amniocentesis: performed at 16 weeks, karyotype after 3 weeks Some laboratories are now able to detect Down's syndrome using fluorescence in-situ hybridization (FISH) in 2 days, but full karyotyping takes 3 weeks Amniocentesis Amniocentesis is a procedure used in prenatal diagnosis. It may be offered after screening tests have indicated a high risk of fetal abnormality or in women considered to be at high risk, for example if > 35 years old. Around 20 ml of fluid is removed by transabdominal needle under ultrasound guidance. Fetal cells present in the amniotic fluid are then studied to aid the diagnosis of a number of conditions. Amniocentesis is usually performed at 16 weeks and the risk of fetal loss is 0.5-1%. The karyotype results typically take 3 weeks. It is known the karyotype may be wrong in 1/1000 cases due to maternal cells being present Conditions which may be diagnosed

1. 2. 3.

neural tube defects (raised AFP levels in the amniotic fluid) chromosomal disorders inborn errors of metabolism

18-

RCGP curriculum

External links RCOG External cephalic version guidelines RCOG Breech presentation guidelines A woman who is 36 weeks pregnant is reviewed. This is her first pregnancy. Her baby is known to currently lie in a breech presentation. What is the most appropriate management? A. Reassure mother baby will most likely turn to a cephalic presentation prior to delivery B. Refer for external cephalic version C. Admit for induction of labour and trial of vaginal delivery D. Refer for radiological pelvimetry E. Admit for caesarean section Breech presentation

In a breech presentation the caudal end of the fetus occupies the lower segment. Whilst around 25% of pregnancies at 28 weeks are breech it only occurs in 3% of babies near term. A frank breech is the most common presentation with the hips flexed and knees fully extended. A footling breech, where one or both feet come first with the bottom at a higher position, is rare but carries a higher perinatal morbidity - Risk factors for breech presentation 1. 2. 3. 4. 5. Uterine malformations, fibroids Placenta praevia Polyhydramnios or Oligohydramnios Fetal abnormality (e.g. CNS malformation, chromosomal disorders) Prematurity (due to increased incidence earlier in gestation)

1. 2.

Cord prolapse is more common in breech presentations Management 1. 2. if < 36 weeks: many fetuses will turn spontaneously if still breech at 36 weeks NICE recommend external cephalic version (ECV)- this has a success rate of around 60%. The RCOG recommend ECV should be offered from 36 weeks in nulliparous women and from 37 weeks in multiparous women if the baby is still breech then delivery options include planned caesarean section or vaginal delivery

3.

Information to help decision making - the RCOG recommend:

1.

2.

'Women should be informed that planned caesarean section carries a reduced perinatal mortality and early neonatal morbidity for babies with a breech presentation at term compared with planned vaginal birth.' 'Women should be informed that there is no evidence that the long term health of babies with a breech presentation delivered at term is influenced by how the baby is born.' 19RCGP curriculum

External links NICE 2008 Antenatal care guidelines A woman who is 28-weeks pregnant presents for antenatal review. She is well and has had no pregnancy related problems to date. Which one of the following is not indicated as part of her routine assessment? A. Urine dipstick B. Blood pressure C. Symphysis-fundal height measurement D. Enquiry about pregnancy related problems E. Auscultation of the fetal heart Routine auscultation for the fetal heart is not recommended by NICE. However, the guidelines suggest that when requested by the mother, auscultation of the fetal heart may provide reassurance.

Antenatal care: timetable NICE issued guidelines on routine care for the healthy pregnant woman in March 2008. They recommend: 1. 2. 3. 10 antenatal visits in the first pregnancy if uncomplicated 7 antenatal visits in subsequent pregnancies if uncomplicated women do not need to be seen by a consultant if the pregnancy is uncomplicated Purpose of visit Booking visit 1. general information e.g. diet, alcohol, smoking, folic acid, vitamin D, antenatal classes BP, urine dipstick, check BMI

Gestation 8 - 12 weeks (ideally < 10 weeks)

2.

Booking bloods/urine 1. 2. 3. 4. FBC, blood group, rhesus status, red cell alloantibodies, haemoglobinopathies hepatitis B, syphilis, rubella HIV test is offered to all women urine culture to detect asymptomatic bacteriuria

10 - 13 weeks

Early scan to confirm dates, exclude multiple pregnancy

11 - 13+6 weeks 16 weeks

Down's syndrome screening including nuchal scan Information on the anomaly and the blood results. If Hb < 11 g/dl consider iron Routine care: BP and urine dipstick Anomaly scan Routine care: BP, urine dipstick, symphysis-fundal height (SFH) Routine care: BP, urine dipstick, SFH Second screen for anaemia and atypical red cell alloantibodies. If Hb < 10.5 g/dl consider iron First dose of anti-D prophylaxis to rhesus negative women Routine care as above

18 - 20+6 weeks 25 weeks (only if primip) 28 weeks

31 weeks (only if primip) 34 weeks

Routine care as above Second dose of anti-D prophylaxis to rhesus negative women Information on labour and birth plan Routine care as above Check presentation - offer external cephalic version if indicated Information on breast feeding, vitamin K, 'baby-

36 weeks

blues' 38 weeks 40 weeks (only if primip) 41 weeks Routine care as above Routine care as above Discussion about options for prolonged pregnancy Routine care as above Discuss labour plans and possibility of induction

20-

RCGP curriculum

External links Fetal Ultrasound Good link on hyperechogenic bowel A woman who is 12 weeks pregnant presents as she is concerned following a recent antenatal scan. The scan has reportedly shown increased nuchal translucency. Other than Down's syndrome, which one of the following is most associated with this finding? A. Renal agenesis B. Cystic fibrosis

C. Polyhydramnios D. Cytomegalovirus infection E. Congenital heart defects Next question A knowledge of fetal abnormalities is specifically mentioned in the Women's Health curriculum statement. Ultrasound in pregnancy A nuchal scan is performed at 11-13 weeks. Causes of an increased nuchal translucency include: 1. 2. 3. Down's syndrome congenital heart defects abdominal wall defects

Causes of hyperechogenic bowel: 1. 2. 3. cystic fibrosis Down's syndrome cytomegalovirus infection 21RCGP curriculum

External links RCOG 2006 Management of severe pre-eclampsia/eclampsia NICE 2010 Hypertension in pregnancy

Which one of the following clinical features would be least consistent with a diagnosis of severe pre-eclampsia? A. Headache B. Epigastric pain C. Reflexes difficult to elicit D. Low platelet count E. Papilloedema Severe pre-eclampsia is associated with hyperreflexia and clonus. A low platelet count may indicate the patient is developing HELLP syndrome Pre-eclampsia Pre-eclampsia is a condition seen after 20 weeks gestation characterised by pregnancy-induced hypertension in association with proteinuria (> 0.3g / 24 hours). Oedema used to be third element of the classic triad but is now often not included in the definition as it is not specific Pre-eclampsia is important as it predisposes to the following problems 1. 2. 3. 4. 5. fetal: prematurity, intrauterine growth retardation eclampsia haemorrhage: placental abruption, intra-abdominal, intracerebral cardiac failure multi-organ failure Risk factors

1. 2. 3. 4. 5. 6. 7. 8. 9.

> 40 years old nulliparity (or new partner) multiple pregnancy body mass index > 30 kg/m^2 diabetes mellitus pregnancy interval of more than 10 years family history of pre-eclampsia previous history of pre-eclampsia pre-existing vascular disease such as hypertension or renal disease

Features of severe pre-eclampsia 1. 2. 3. 4. 5. 6. 7. 8. hypertension: typically > 170/110 mmHg and proteinuria as above proteinuria: dipstick ++/+++ headache visual disturbance papilloedema RUQ/epigastric pain hyperreflexia platelet count < 100 * 106/l, abnormal liver enzymes or HELLP syndrome

Management 1. consensus guidelines recommend treating blood pressure > 160/110 mmHg although many clinicians have a lower threshold oral labetalol is now first-line following the 2010 NICE guidelines. Nifedipine and hydralazine may also be used delivery of the baby is the most important and definitive management step. The timing depends on the individual clinical scenario

2. 3.

22-

RCGP curriculum

3.06 - Womens Health External links RCOG Chickenpox in pregnancy guidelines Department of Health Green-top guidelines A woman who is 14 weeks pregnant presents as she came into contact with a child who has chickenpox around 4 days ago. She is unsure if she had the condition herself as a child. Blood tests show the following: Varicella IgM Negative Varicella IgG Negative What is the most appropriate management? A. Varicella zoster immunoglobulin B. No action required C. IV aciclovir D. Varicella zoster vaccination E. Varicella zoster vaccination + varicella zoster

immunoglobulin

Chickenpox exposure in pregnancy - if not immune give VZIG The negative IgG indicates no previous exposure to chickenpox Chickenpox exposure in pregnancy Chickenpox is caused by primary infection with varicella zoster virus. Shingles is reactivation of dormant virus in dorsal root ganglion. In pregnancy there is a risk to both the mother and also the fetus, a syndrome now termed fetal varicella syndrome Fetal varicella syndrome (FVS) 1. 2. risk of FVS following maternal varicella exposure is around 1% if occurs before 20 weeks gestation studies have shown a very small number of cases occurring between 20-28 weeks gestation and none following 28 weeks features of FVS include skin scarring, eye defects (microphthalmia), limb hypoplasia, microcephaly and learning disabilities

3.

Management of chickenpox exposure 1. if there is any doubt about the mother previously having chickenpox maternal blood should be checked for varicella antibodies if the pregnant women is not immune to varicella she should be given varicella zoster immunoglobulin (VZIG) as soon as possible. RCOG and Greenbook guidelines suggest VZIG is effective up to 10 days post exposure

2.

3.

consensus guidelines suggest oral aciclovir should be given if pregnant women with chickenpox present within 24 hours of onset of the rash 23RCGP curriculum

3.06 - Womens Health External links NICE 2008 Antenatal care guidelines A 27-year-old woman has just found out she is pregnant. There is no past medical history of note and this is her first pregnancy. She asks for advice about vitamin D supplementation. What is the most appropriate advice to give? A. Offer vitamin D supplementation if she feels her diet is inadequate B. Should be avoided - potential risk to developing fetus C. Await booking bloods for confirmation of vitamin D levels D. Advise to increase milk and soft-cheese consumption e.g. Brie and Stilton E. Advise her she can take supplements if she wishes but this is not part of routine NHS antenatal advice

Soft-cheese should be avoided during pregnancy due to the risk of Listeria Antenatal care: specific points NICE issued guidelines on routine care for the healthy pregnant woman in March 2008 Nausea and vomiting 1. 2. natural remedies - ginger and acupuncture on the 'p6' point (by the wrist) are recommended by NICE antihistamines should be used first-line (BNF suggests promethazine as first-line)

Vitamin D 1. NICE recommend 'All women should be informed at the booking appointment about the importance for their own and their baby's health of maintaining adequate vitamin D stores during pregnancy and whilst breastfeeding' 'women may choose to take 10 micrograms of vitamin D per day, as found in the Healthy Start multivitamin supplement' particular care should be taken with women at risk (e.g. Asian, obese, poor diet)

2. 3.

Alcohol 1. 2. NICE recommend women should avoid alcohol during the first trimester if women choose to drink alcohol during pregnancy they should be advised to drink no more than 1 to 2 UK units once or twice a week 24RCGP curriculum

3.06 - Womens Health The chance of a 45-year-old mother giving birth to a child with Down's syndrome is approximately:

A. 1 in 5 B. 1 in 10 C. 1 in 30 D. 1 in 100 E. 1 in 500

Down's syndrome risk - 1/1,000 at 30 years then divide by 3 for every 5 years Down's syndrome: epidemiology and genetics Risk of Down's syndrome with increasing maternal age 1. 2. 3. 4. risk at 30 years = 1/1000 35 years = 1/350 40 years = 1/100 45 years = 1/30

One way of remembering this is by starting at 1/1,000 at 30 years and then dividing the denominator by 3 (i.e. 3 times more common) for every extra 5 years of age Cytogenetics: Mode % of cases Risk of recurrence

Non-disjunction

94%

1 in 100 if under mother < 35 years 10-15% if mother is translocation carrier 2.5% if father is translocation carrier

Robertsonian translocation (usually onto 14)

5%

Mosaicism

1%

The chance of a further child with Down's syndrome is approximately 1 in 100 if the mother is less than 35 years old. If the trisomy 21 is a result of a translocation the risk is much higher

25-

RCGP curriculum

3.06 - Womens Health External links NICE 2008 Antenatal care guidelines What is the current recommended antenatal screening tests for Down's syndrome in the UK? A. B-HCG + alpha-feto protein + oestradiol B. Nuchal translucency + B-HCG + alpha-feto protein

C. B-HCG + alpha-feto protein D. B-HCG + pregnancy associated plasma protein A E. Nuchal translucency + B-HCG + pregnancy associated plasma protein A

Down's - antenatal screening: Nuchal translucency + B-HCG + pregnancy associated plasma protein A Down's syndrome: antenatal testing NICE issued guidelines on antenatal care in March 2008 including advice on screening for Down's syndrome: 1. the combined test is now standard: nuchal translucency measurement + serum B-HCG + pregnancy associated plasma protein A these tests should be done between 11 - 13+6 weeks if women book later in pregnancy either the triple* or quadruple test** should be offered between 15 - 20 weeks

2. 3.

*alpha-fetoprotein, unconjugated oestriol, human chorionic gonadotrophin **alpha-fetoprotein, unconjugated oestriol, human chorionic gonadotrophin and inhibin-A 26External links

Australian goverment Prescribing in pregnancy

A 39-year-old woman who has recently been diagnosed with hypertension comes for review. She is sexually active but does not currently use any form of contraception other than barrier methods. Which one of the following medications should be avoided? A. Hydralazine B. Nifedipine C. Methyldopa D. Labetalol E. Lisinopril When prescribing this woman should be treated as if she were pregnant given the absence of effective contraception. ACE inhibitors such as lisinopril are known teratogens and most be avoided. Prescribing in pregnant patients Very few drugs are known to be completely safe in pregnancy. The list below largely comprises of those known to be harmful. Some countries have developed a grading system - see the link. Antibiotics 1. 2. 3. 4. tetracyclines aminoglycosides sulphonamides and trimethoprim quinolones: the BNF advises to avoid due to arthropathy in some animal studies

Other drugs

1. 2. 3. 4. 5. 6.

ACE inhibitors, angiotensin II receptor antagonists statins warfarin sulfonylureas retinoids (including topical) cytotoxic agents

The majority of antiepileptics including valproate, carbamazepine and phenytoin are known to be potentially harmful. The decision to stop such treatments however is difficult as uncontrolled epilepsy is also a risk 27RCGP curriculum

3.06 - Womens Health External links NICE 2010 Hypertension in pregnancy Which one of the following statements is not correct regarding hypertension in pregnancy? A. An increase above booking readings of > 30 mmHg systolic or > 15 mmHg diastolic suggests hypertension B. Pre-eclampsia occurs in around 5% of pregnancies C. Urine dipstick showing protein + is consistent with gestational hypertension D. A rise in blood pressure before 20 weeks suggests

pre-existing hypertension E. With gestational hypertension the blood pressure rises in the second half of pregnancy Proteinuria suggests pre-eclampsia Hypertension in pregnancy The classification of hypertension in pregnancy is complicated and varies. Remember, in normal pregnancy: 1. 2. blood pressure usually falls in the first trimester (particularly the diastolic), and continues to fall until 20-24 weeks after this time the blood pressure usually increases to prepregnancy levels by term

Hypertension in pregnancy in usually defined as: 1. 2. systolic > 140 mmHg or diastolic > 90 mmHg or an increase above booking readings of > 30 mmHg systolic or > 15 mmHg diastolic

After establishing that the patient is hypertensive they should be categorised into one of the following groups Pre-existing hypertension Pregnancy-induced hypertension (PIH, also known as gestational hypertension) Hypertension (as defined above) occurring Pre-eclampsia

A history of hypertension before

Pregnancy-induced hypertension in

pregnancy or an elevated blood pressure > 140/90 mmHg before 20 weeks gestation No proteinuria, no oedema Occurs in 3-5% of pregnancies and is more common in older women

in the second half of pregnancy (i.e. after 20 weeks) No proteinuria, no oedema Occurs in around 5-7% of pregnancies Resolves following birth (typically after one month). Women with PIH are at increased risk of future pre-eclampsia or hypertension later in life

association with proteinuria (> 0.3g / 24 hours) Oedema may occur but is now less commonly used as a criteria Occurs in around 5% of pregnancies

28-

RCGP curriculum

3.06 - Womens Health A woman who is 34 weeks pregnant is found to have an amniotic fluid volume of 440 ml. Which one of the following conditions is not part of the differential diagnosis? A. Premature rupture of membranes B. Pre-eclampsia C. Tracheo-oesophageal fistula

D. Renal agenesis E. Intrauterine growth restriction An amniotic fluid volume of 440ml indicates oligohydramnios. Tracheo-oesophageal fistula is associated with polyhydramnios. Oligohydramnios In oligohydramnios there is reduced amniotic fluid. Definitions vary but include less than 500ml at 32-36 weeks and an amniotic fluid index (AFI) < 5th percentile. Causes: 1. 2. 3. 4. 5. premature rupture of membranes fetal renal problems e.g. renal agenesis intrauterine growth restriction post-term gestation pre-eclampsia 29RCGP curriculum

3.06 - Womens Health External links NICE 2008 Management of diabetes in pregnancy SIGN Management of diabetes in pregnancy Postgraduate Medical Journal DKA in pregnancy A 29-year-old woman has just found out she is pregnant for the second time. Her first pregnancy was complicated by gestational

diabetes. Following her first pregnancy she was told she was no longer diabetic. What is the most appropriate management? A. Start insulin B. Start metformin and do oral glucose tolerance test at 12-14 weeks C. Do oral glucose tolerance test at booking visit D. Do oral glucose tolerance test at 16-18 weeks E. Do fasting glucose at booking visit Pregnancy: diabetes mellitus Diabetes mellitus may be a pre-existing problem or develop during pregnancy, gestational diabetes. It complicates around 1 in 40 pregnancies Risk factors for gestational diabetes 1. 2. 3. 4. 5. BMI of > 30 kg/m^2 previous macrosomic baby weighing 4.5 kg or above. previous gestational diabetes first-degree relative with diabetes family origin with a high prevalence of diabetes (South Asian, black Caribbean and Middle Eastern) Screening for gestational diabetes 1. if a women has had gestational diabetes previously an oral glucose tolerance test (OGTT) should be performed at 16-18 weeks and at 28 weeks if the first test is normal

2. 3.

women with any of the other risk factors should be offered an OGTT at 24-28 weeks currently the same WHO diagnostic criteria are used as for non-pregnant patients. There is however increasing evidence that a lower threshold should be used as treating borderline patients improves both maternal and neonatal outcomes

NICE issued guidelines on the management of diabetes mellitus in pregnancy in 2008 Management of pre-existing diabetes 1. 2. 3. 4. 5. 6. weight loss for women with BMI of > 27 kg/m^2 stop oral hypoglycaemic agents, apart from metformin, and commence insulin folic acid 5 mg/day from pre-conception to 12 weeks gestation detailed anomaly scan at 18-20 weeks including fourchamber view of the heart and outflow tracts tight glycaemic control reduces complication rates treat retinopathy as can worsen during pregnancy

Management of gestational diabetes 1. 2. responds to changes in diet and exercise in around 80% of women oral hypoglycaemic agents (metformin or glibenclamide) or insulin injections are needed if blood glucose control is poor or this is any evidence of complications (e.g. macrosomia) there is increasing evidence that oral hypoglycaemic agents are both safe and give similar outcomes to insulin hypoglycaemic medication should be stopped following delivery a fasting glucose should be checked at the 6 week postnatal check

3. 4. 5.

30-

Question stats

A new mother who is 4 weeks post-partum presents for review. She has developed a warm, red tender patch to on the right breast just lateral to the areola. This has been getting worse for the past three days and feeding is now painful. She saw the midwife yesterday who helped with positioning but this has not improved matters. On examination she has mastitis of the right breast with no obvious abscess. What is the most appropriate management? A. Co-amoxiclav for 7 days, continue breast feeding B. Flucloxacillin for 14 days, continue breast feeding C. Flucloxacillin for 7 days, stop breast feeding D. Co-amoxiclav for 7 days, stop breast feeding E. Clindamycin for 7 days, continue breast feeding Breast feeding problems Mastitis Mastitis affects around 1 in 10 breast feeding women. The BNF advises to treat 'if systemically unwell, if nipple fissure present, if symptoms do not improve after 12-24 hours of effective milk removal of if culture indicates infection'. The first-line antibiotic is flucloxacillin for 10-14 days. Breast feeding or expressing should continue during treatment. If left untreated, mastitis may develop into a breast abscess. This generally requires incision and drainage.

31-

RCGP curriculum

3.06 - Womens Health A 23-year-old woman who is 10 weeks pregnant presents with severe vomiting. She is now having difficulty keeping down fluids and a dipstick of her urine shows ketones ++. Which one of the following is not associated with an increased risk of this condition? A. Obesity B. Trophoblastic disease C. Smoking D. Nulliparity E. Carrying twins

Smoking is associated with a decreased incidence of hyperemesis gravidarum

Hyperemesis gravidarum Hyperemesis gravidarum describes excessive vomiting during pregnancy. It occurs in around 1% of pregnancies and is thought to be related to raised beta hCG levels. Hyperemesis gravidarum

is most common between 8 and 12 weeks but may persist up to 20 weeks*. Associations

Multiple pregnancies Trophoblastic disease Hyperthyroidism Nulliparity Obesity

Smoking is associated with a decreased incidence of hyperemesis Management


Antihistamines should be used first-line (BNF suggests promethazine as first-line) Admission may be needed for IV hydration

Complications

Wernicke's encephalopathy Mallory-Weiss tear Central pontine myelinolysis Acute tubular necrosis Fetal: small for gestational age, pre-term birth

*And in very rare cases beyond 20 weeks

dr.naila@yahoo.com - Applied Knowledge Test - My account

32-

RCGP curriculum

3.06 - Womens Health Search Go A pregnant woman has serum alpha feto-protein levels measured. Which one of the following is associated with a low alpha-feto protein level? A. Meningocele B. Gastroschisis C. Multiple pregnancy D. Down's syndrome E. Anencephaly

AFP

Raised in neural tubes defects Decreased in Down's syndrome

Alpha feto-protein Alpha-fetoprotein (AFP) is a protein produced by the developing fetus

Increased AFP Neural tube defects (meningocele, myelomeningocele and anencephaly) Abdominal wall defects (omphalocele and gastroschisis) Multiple pregnancy

Decreased AFP Down's syndrome Trisomy 18 Maternal diabetes mellitus

dr.naila@yahoo.com - Applied Knowledge Test - My account 33RCGP curriculum

3.06 - Womens Health Search Go External links Civil Aviation Authority Fitness to fly guidelines A pregnant woman asks for advice about flying. What is the latest time in her pregnancy that she may fly, presuming an uncomplicated pregnancy with no change in the estimated date of delivery? A. 24 weeks

B. 28 weeks C. 32 weeks D. 36 weeks E. 38 weeks

Fitness to fly - pregnancy - up to 36 weeks

Fitness to fly The Civil Aviation Authority (CAA) has issued guidelines on air travel for people with medical conditions; please see the link provided. Cardiovascular disease

unstable angina, uncontrolled hypertension, uncontrolled cardiac arrhythmia, decompensated heart failure, severe symptomatic valvular disease: should not fly uncomplicated myocardial infarction: may fly after 7-10 days complicated myocardial infarction: after 4-6 weeks coronary artery bypass graft: after 10-14 days percutaneous coronary intervention: after 5 days

Respiratory disease

pneumonia: should be 'clinically improved with no residual infection' pneumothorax: absolute contraindication, the CAA suggest patients may travel 2 weeks after successful drainage if there is no residual air. The British Thoracic Society used to recommend not travelling by air for a period of 6 weeks but this has now been changed to 1 week post check x-ray

Pregnancy

most airlines do not allow travel after 36 weeks for a single pregnancy and after 32 weeks for a multiple pregnancy most airlines require a certificate after 28 weeks confirming that the pregnancy is progressing normally

Surgery

travel should be avoided for 10 days following abdominal surgery laparoscopic surgery: after 24 hours colonoscopy: after 24 hours following the application of a plaster cast, the majority of airlines restrict flying for 24 hours on flights of less than 2 hours or 48 hours for longer flights

Haematological disorders

patients with a haemoglobin of greater than 8 g/dl may travel without problems (assuming there is no coexisting condition such as cardiovascular or respiratory disease) dr.naila@yahoo.com - Applied Knowledge Test - My account

34-

RCGP curriculum

3.06 - Womens Health Search Go External links RCOG 2006 Management of severe pre-eclampsia/eclampsia NICE 2010 Hypertension in pregnancy A 38-year-old female develops hypertension in the third trimester of her first pregnancy. A 24 hour urine collection shows 0.5g protein. Which one of the following complications is least associated with this condition? A. Intracerebral haemorrhage B. Pulmonary oedema C. Fetal prematurity D. Transverse myelitis E. Fetal intrauterine growth retardation Transverse myelitis is not associated with pre-eclampsia Pre-eclampsia

Pre-eclampsia is a condition seen after 20 weeks gestation characterised by pregnancy-induced hypertension in association with proteinuria (> 0.3g / 24 hours). Oedema used to be third element of the classic triad but is now often not included in the definition as it is not specific Pre-eclampsia is important as it predisposes to the following problems

fetal: prematurity, intrauterine growth retardation eclampsia haemorrhage: placental abruption, intra-abdominal, intracerebral cardiac failure multi-organ failure

Risk factors

> 40 years old nulliparity (or new partner) multiple pregnancy body mass index > 30 kg/m^2 diabetes mellitus pregnancy interval of more than 10 years family history of pre-eclampsia previous history of pre-eclampsia pre-existing vascular disease such as hypertension or renal disease

Features of severe pre-eclampsia

hypertension: typically > 170/110 mmHg and proteinuria as above

proteinuria: dipstick ++/+++ headache visual disturbance papilloedema RUQ/epigastric pain hyperreflexia platelet count < 100 * 106/l, abnormal liver enzymes or HELLP syndrome

Management

consensus guidelines recommend treating blood pressure > 160/110 mmHg although many clinicians have a lower threshold oral labetalol is now first-line following the 2010 NICE guidelines. Nifedipine and hydralazine may also be used delivery of the baby is the most important and definitive management step. The timing depends on the individual clinical scenario

dr.naila@yahoo.com - Applied Knowledge Test - My account 35RCGP curriculum

3.06 - Womens Health Search Go External links NICE 2008 Antenatal care guidelines

According to recent NICE guidelines, which one of the following tests should not be offered, as a matter of routine, to women during the booking visit for antenatal care? A. Syphilis B. HIV C. Blood glucose D. Hepatitis B E. Screening for haemoglobinopathies

Antenatal care: routine glucose testing no longer recommended

Surprisingly perhaps, NICE now recommends that blood glucose is only checked to those considered at risk (e.g. obesity, previous macrosomic baby, family history, Asian ethnicity) Antenatal care: timetable NICE issued guidelines on routine care for the healthy pregnant woman in March 2008. They recommend:

10 antenatal visits in the first pregnancy if uncomplicated 7 antenatal visits in subsequent pregnancies if uncomplicated women do not need to be seen by a consultant if the pregnancy is uncomplicated

Gestation 8 - 12 weeks (ideally < 10 weeks)

Purpose of visit Booking visit

general information e.g. diet, alcohol, smoking, folic acid, vitamin D, antenatal classes BP, urine dipstick, check BMI

Booking bloods/urine

FBC, blood group, rhesus status, red cell alloantibodies, haemoglobinopathies hepatitis B, syphilis, rubella HIV test is offered to all women urine culture to detect asymptomatic bacteriuria

10 - 13+6 weeks

Early scan to confirm dates, exclude multiple pregnancy Down's syndrome screening including nuchal scan Information on the anomaly and the blood results. If Hb < 11 g/dl consider iron Routine care: BP and urine dipstick Anomaly scan

11 - 13+6 weeks 16 weeks

18 - 20+6 weeks

25 weeks (only if primip) 28 weeks

Routine care: BP, urine dipstick, symphysis-fundal height (SFH) Routine care: BP, urine dipstick, SFH Second screen for anaemia and atypical red cell alloantibodies. If Hb < 10.5 g/dl consider iron First dose of anti-D prophylaxis to rhesus negative women Routine care as above

31 weeks (only if primip) 34 weeks

Routine care as above Second dose of anti-D prophylaxis to rhesus negative women* Information on labour and birth plan Routine care as above Check presentation - offer external cephalic version if indicated Information on breast feeding, vitamin K, 'babyblues' Routine care as above Routine care as above Discussion about options for prolonged pregnancy

36 weeks

38 weeks 40 weeks (only if primip)

41 weeks

Routine care as above Discuss labour plans and possibility of induction

*The evidence base suggests that there is little difference in the efficacy of single-dose (at 28 weeks) and double-dose regimes (at 28 & 34 weeks). For this reason the RCOG in 2011 advised that either regime could be used 'depending on local factors' dr.naila@yahoo.com - Applied Knowledge Test - My account 36RCGP curriculum

3.06 - Womens Health Search Go A 29-year-old female who is 14 weeks into her first pregnancy is investigated for excessive sweating and tremor. Blood tests reveal the following: TSH < 0.05 mu/l T4 188 nmol/l

What is the most appropriate management? A. Immediate surgery B. Carbimazole + thyroxine

C. Surgery at start of third trimester D. Propylthiouracil E. Radioiodine Propylthiouracil is traditionally taught as the antithyroid drug of choice in pregnancy. This approach was supported by the 2007 Endocrine Society consensus guidelines. It also has the advantage of being excreted to a lesser extent than carbimazole in breast milk. Despite this some endocrinologists use carbimazole and the BNF states both drugs may be used in pregnancy. Carbimazole has rarely been associated with aplasia cutis of the neonate Pregnancy: thyroid problems In pregnancy there is an increase in the levels of thyroxinebinding globulin (TBG). This causes an increase in the levels of total thyroxine but does not affect the free thyroxine level Thyrotoxicosis Untreated thyrotoxicosis increases the risk of fetal loss, maternal heart failure and premature labour Graves' disease is the most common cause of thyrotoxicosis in pregnancy. It is also recognised that activation of the TSH receptor by HCG may also occur - often termed transient gestational hyperthyroidism. HCG levels will fall in second and third trimester

Management

propylthiouracil has traditionally been the antithyroid drug of choice. This approach was supported by the 2007 Endocrine Society consensus guidelines maternal free thyroxine levels should be kept in the upper third of the normal reference range to avoid fetal hypothyroidism thyrotrophin receptor stimulating antibodies should be checked at 30-36 weeks gestation - helps to determine risk of neonatal thyroid problems block-and-replace regimes should not be used in pregnancy radioiodine therapy is contraindicated

Hypothyroidism Key points


thyroxine is safe during pregnancy serum thyroid stimulating hormone measured in each trimester and 6-8 weeks post-partum some women require an increased dose of thyroxine during pregnancy breast feeding is safe whilst on thyroxine

dr.naila@yahoo.com - Applied Knowledge Test - My account 37RCGP curriculum

3.06 - Womens Health Search

Go External links NICE 2008 Antenatal care guidelines A pregnant woman asks for advice about alcohol consumption during pregnancy. Which one of the following is in line with current NICE guidelines? A. 1 to 2 units once or twice per week throughout pregnancy B. Avoid first and second trimester. If then chooses to drink 1 to 4 units no more than twice per week C. 1 to 2 units once per week throughout pregnancy D. Avoid first trimester. If then chooses to drink 1 to 2 units once or twice per week E. Avoid first trimester. If then chooses to drink less than 7-14 units per week Antenatal care: specific points NICE issued guidelines on routine care for the healthy pregnant woman in March 2008 Nausea and vomiting

Natural remedies - ginger and acupuncture on the 'p6' point (by the wrist) are recommended by NICE

Antihistamines should be used first-line (BNF suggests promethazine as first-line)

Vitamin D

NICE recommend 'All women should be informed at the booking appointment about the importance for their own and their baby's health of maintaining adequate vitamin D stores during pregnancy and whilst breastfeeding' 'Women may choose to take 10 micrograms of vitamin D per day, as found in the Healthy Start multivitamin supplement' Particular care should be taken with women at risk (e.g. Asian, obese, poor diet)

Alcohol

NICE recommend women should avoid alcohol during the first trimester If women choose to drink alcohol during pregnancy they should be advised to drink no more than 1 to 2 UK units once or twice a week dr.naila@yahoo.com - Applied Knowledge Test - My account

38-

RCGP curriculum

3.06 - Womens Health Search Go External links

NICE 2010 Hypertension in pregnancy A 37-year-old woman presents for review. She is 26 weeks pregnant and has had no problems with her pregnancy to date. Blood pressure is 144/92 mmHg, a rise from her booking reading of 110/80 mmHg. Urine dipstick reveals the following: Protein negative

Leucocytes negative Blood negative

What is the most appropriate description of her condition? A. Moderate pre-eclampsia B. Mild pre-eclampsia C. Gestational hypertension D. Normal physiological change in blood pressure E. Pre-existing hypertension Hypertension in pregnancy The classification of hypertension in pregnancy is complicated and varies. Remember, in normal pregnancy:

Blood pressure usually falls in the first trimester (particularly the diastolic), and continues to fall until 20-24 weeks

After this time the blood pressure usually increases to prepregnancy levels by term

Hypertension in pregnancy in usually defined as:


Systolic > 140 mmHg or diastolic > 90 mmHg Or an increase above booking readings of > 30 mmHg systolic or > 15 mmHg diastolic

After establishing that the patient is hypertensive they should be categorized into one of the following groups Pre-existing hypertension Pregnancy-induced hypertension (PIH, also known as gestational hypertension) Hypertension (as defined above) occurring in the second half of pregnancy (i.e. after 20 weeks) No proteinuria, no edema Occurs in around 5-7% of pregnancies Resolves following birth Pre-eclampsia

A history of hypertension before pregnancy or an elevated blood pressure > 140/90 mmHg before 20 weeks gestation No proteinuria, no edema Occurs in 3-5% of pregnancies and is more common in older

Pregnancy-induced hypertension in association with proteinuria (> 0.3g / 24 hours) Edema may occur but is now less commonly used as a criteria Occurs in around 5% of pregnancies

women

(typically after one month). Women with PIH are at increased risk of future pre-eclampsia or hypertension later in life

dr.naila@yahoo.com - Applied Knowledge Test - My account 39RCGP curriculum

3.06 - Womens Health Search Go External links NICE 2008 Management of diabetes in pregnancy SIGN Management of diabetes in pregnancy Postgraduate Medical Journal DKA in pregnancy You are counselling a 23-year-old woman with type 1 diabetes mellitus who has just found out she is pregnant. She is concerned about the increased risk congenital malformations, particularly heart defects. Which one of the following best describes the screening that is currently offered?

A. No extra screening is offered B. Echocardiography at 6 weeks C. Scan at 34-36 weeks including four-chamber view of the heart and outflow tracts D. Echocardiography in the first week of life E. Scan at 18-20 weeks including four-chamber view of the heart and outflow tracts

Diabetes in pregnancy: detailed heart scan at 18-20 weeks

Pregnancy: diabetes mellitus Diabetes mellitus may be a pre-existing problem or develop during pregnancy, gestational diabetes. It complicates around 1 in 40 pregnancies Risk factors for gestational diabetes

BMI of > 30 kg/m^2 previous macrosomic baby weighing 4.5 kg or above. previous gestational diabetes first-degree relative with diabetes family origin with a high prevalence of diabetes (South Asian, black Caribbean and Middle Eastern)

Screening for gestational diabetes

if a women has had gestational diabetes previously an oral glucose tolerance test (OGTT) should be performed at 16-18 weeks and at 28 weeks if the first test is normal women with any of the other risk factors should be offered an OGTT at 24-28 weeks currently the same WHO diagnostic criteria are used as for non-pregnant patients. There is however increasing evidence that a lower threshold should be used as treating borderline patients improves both maternal and neonatal outcomes

NICE issued guidelines on the management of diabetes mellitus in pregnancy in 2008 Management of pre-existing diabetes

weight loss for women with BMI of > 27 kg/m^2 stop oral hypoglycaemic agents, apart from metformin, and commence insulin folic acid 5 mg/day from pre-conception to 12 weeks gestation detailed anomaly scan at 18-20 weeks including fourchamber view of the heart and outflow tracts tight glycaemic control reduces complication rates treat retinopathy as can worsen during pregnancy

Management of gestational diabetes

responds to changes in diet and exercise in around 80% of women

oral hypoglycaemic agents (metformin or glibenclamide) or insulin injections are needed if blood glucose control is poor or this is any evidence of complications (e.g. macrosomia) there is increasing evidence that oral hypoglycaemic agents are both safe and give similar outcomes to insulin hypoglycaemic medication should be stopped following delivery a fasting glucose should be checked at the 6 week postnatal check

dr.naila@yahoo.com - Applied Knowledge Test - My account 40RCGP curriculum

3.06 - Womens Health Search Go External links NICE 2008 Routine anti-D prophylaxis guidelines RCOG Use of Anti-D Immunoglobulin for Rh Prophylaxis In which one of the following situations would you not routinely give anti-D prophylaxis to a non-sensitised Rhesus negative mother? A. Amniocentesis at 16 weeks

B. External cephalic version at 37 weeks C. Spontaneous miscarriage at 10 weeks D. Placental abruption at 26 weeks E. Ectopic pregnancy at 7 weeks Rhesus negative pregnancy A basic understanding of the pathophysiology is essential to understand the management of Rhesus negative pregnancies

along with the ABO system the Rhesus system is the most important antigen found on red blood cells. The D antigen is the most important antigen of the rhesus system around 15% of mothers are rhesus negative (Rh -ve) if a Rh -ve mother delivers a Rh +ve child a leak of fetal red blood cells may occur this causes anti-D IgG antibodies to form in mother in later pregnancies these can cross placenta and cause haemolysis in fetus this can also occur in the first pregnancy due to leaks

Prevention

test for D antibodies in all Rh -ve mothers at booking NICE (2008) advise giving anti-D to Rh -ve mothers at 28 and 34 weeks the evidence base suggests that there is little difference in the efficacy of single-dose (at 28 weeks) and double-dose regimes (at 28 & 34 weeks). For this reason the RCOG in

2011 advised that either regime could be used 'depending on local factors' anti-D is prophylaxis - once sensitization has occurred it is irreversible if event is in 2nd/3rd trimester give large dose of anti-D and perform Kleihauer test - determines proportion of fetal RBCs present

Anti-D immunoglobulin should be given as soon as possible (but always within 72 hours) in the following situations:

delivery of a Rh +ve infant, whether live or stillborn any termination of pregnancy miscarriage if gestation is > 12 weeks ectopic pregnancy external cephalic version antepartum haemorrhage amniocentesis, chorionic villus sampling, fetal blood sampling

Tests

all babies born to Rh -ve mother should have cord blood taken at delivery for FBC, blood group & direct Coombs test Coombs test: direct antiglobulin , will demonstrate antibodies on RBCs of baby Kleihauer test: add acid to maternal blood, fetal cells are resistant

Affected fetus

oedematous (hydrops fetalis, as liver devoted to RBC production albumin falls)

jaundice, anaemia, hepatosplenomegaly heart failure kernicterus treatment: transfusions, UV phototherapy

dr.naila@yahoo.com - Applied Knowledge Test - My account 41RCGP curriculum

3.06 - Womens Health Search Go External links NICE 2008 Antenatal care guidelines Theme: Routine antenatal care A. 7 - 8 weeks B. 8 - 12 weeks C. 11 - 13+6 weeks D. 14 -15 weeks E. 16 weeks F. 18 - 20+6 weeks G. 25 weeks

H. 28 weeks I. 34 weeks J. 38 weeks For each of the following components of routine antenatal care select the gestation when it should occur 10. Second screen for anaemia and atypical red cell alloantibodies 28 weeks 11. Nuchal scan 11 - 13+6 weeks 12. Urine culture to detect asymptomatic bacteriuria 8 - 12 weeks Antenatal care: timetable

NICE issued guidelines on routine care for the healthy pregnant woman in March 2008. They recommend:

10 antenatal visits in the first pregnancy if uncomplicated 7 antenatal visits in subsequent pregnancies if uncomplicated

women do not need to be seen by a consultant if the pregnancy is uncomplicated

Gestation 8 - 12 weeks (ideally < 10 weeks)

Purpose of visit Booking visit

general information e.g. diet, alcohol, smoking, folic acid, vitamin D, antenatal classes BP, urine dipstick, check BMI

Booking bloods/urine

FBC, blood group, rhesus status, red cell alloantibodies, haemoglobinopathies hepatitis B, syphilis, rubella HIV test is offered to all women urine culture to detect asymptomatic bacteriuria

10 - 13+6 weeks

Early scan to confirm dates, exclude multiple pregnancy Down's syndrome screening including nuchal scan Information on the anomaly and the blood results. If Hb < 11 g/dl consider iron Routine care: BP and urine dipstick

11 - 13+6 weeks 16 weeks

18 - 20+6 weeks 25 weeks (only if primip) 28 weeks

Anomaly scan Routine care: BP, urine dipstick, symphysis-fundal height (SFH) Routine care: BP, urine dipstick, SFH Second screen for anaemia and atypical red cell alloantibodies. If Hb < 10.5 g/dl consider iron First dose of anti-D prophylaxis to rhesus negative women Routine care as above

31 weeks (only if primip) 34 weeks

Routine care as above Second dose of anti-D prophylaxis to rhesus negative women* Information on labour and birth plan Routine care as above Check presentation - offer external cephalic version if indicated Information on breast feeding, vitamin K, 'babyblues' Routine care as above Routine care as above

36 weeks

38 weeks 40 weeks (only if

primip) 41 weeks

Discussion about options for prolonged pregnancy Routine care as above Discuss labour plans and possibility of induction

*the evidence base suggests that there is little difference in the efficacy of single-dose (at 28 weeks) and double-dose regimes (at 28 & 34 weeks). For this reason the RCOG in 2011 advised that either regime could be used 'depending on local factors' dr.naila@yahoo.com - Applied Knowledge Test - My account 42RCGP curriculum

3.06 - Womens Health Search Go External links NICE 2008 Antenatal care guidelines A 23-year-old woman attends her antenatal booking appointment. She thinks that she is 10 weeks pregnant. This is her first pregnancy. Which one of the following is not routinely performed? A. Assess body mass index B. Pelvic examination

C. Urine culture if dipstick urine normal D. Check for red cell alloantibodies E. Hepatitis B testing Antenatal care: timetable NICE issued guidelines on routine care for the healthy pregnant woman in March 2008. They recommend:

10 antenatal visits in the first pregnancy if uncomplicated 7 antenatal visits in subsequent pregnancies if uncomplicated women do not need to be seen by a consultant if the pregnancy is uncomplicated

Gestation 8 - 12 weeks (ideally < 10 weeks)

Purpose of visit Booking visit

general information e.g. diet, alcohol, smoking, folic acid, vitamin D, antenatal classes BP, urine dipstick, check BMI

Booking bloods/urine

FBC, blood group, rhesus status, red cell alloantibodies, haemoglobinopathies

hepatitis B, syphilis, rubella HIV test is offered to all women urine culture to detect asymptomatic bacteriuria

10 - 13+6 weeks

Early scan to confirm dates, exclude multiple pregnancy Down's syndrome screening including nuchal scan Information on the anomaly and the blood results. If Hb < 11 g/dl consider iron Routine care: BP and urine dipstick Anomaly scan Routine care: BP, urine dipstick, symphysis-fundal height (SFH) Routine care: BP, urine dipstick, SFH Second screen for anaemia and atypical red cell alloantibodies. If Hb < 10.5 g/dl consider iron First dose of anti-D prophylaxis to rhesus negative women Routine care as above

11 - 13+6 weeks 16 weeks

18 - 20+6 weeks 25 weeks (only if primip) 28 weeks

31 weeks (only if primip)

34 weeks

Routine care as above Second dose of anti-D prophylaxis to rhesus negative women* Information on labour and birth plan Routine care as above Check presentation - offer external cephalic version if indicated Information on breast feeding, vitamin K, 'babyblues' Routine care as above Routine care as above Discussion about options for prolonged pregnancy Routine care as above Discuss labour plans and possibility of induction

36 weeks

38 weeks 40 weeks (only if primip) 41 weeks

*the evidence base suggests that there is little difference in the efficacy of single-dose (at 28 weeks) and double-dose regimes (at 28 & 34 weeks). For this reason the RCOG in 2011 advised that either regime could be used 'depending on local factors' dr.naila@yahoo.com - Applied Knowledge Test - My account 43RCGP curriculum

3.06 - Womens Health Search

Go External links NICE 2008 Routine anti-D prophylaxis guidelines RCOG Use of Anti-D Immunoglobulin for Rh Prophylaxis Which one of the following statements regarding the management of pregnant, non-sensitised Rhesus negative women is incorrect? A. External cephalic version does not require prophylaxis B. Around 15% of mothers are Rhesus negative C. Hydrops fetalis may result if maternal anti-D antibodies cross the placenta D. All Rhesus negative women should receive anti-D at 28 and 34 weeks E. Anti-D should be given following every termination of pregnancy Rhesus negative pregnancy A basic understanding of the pathophysiology is essential to understand the management of Rhesus negative pregnancies

along with the ABO system the Rhesus system is the most important antigen found on red blood cells. The D antigen is the most important antigen of the rhesus system around 15% of mothers are rhesus negative (Rh -ve) if a Rh -ve mother delivers a Rh +ve child a leak of fetal red blood cells may occur this causes anti-D IgG antibodies to form in mother in later pregnancies these can cross placenta and cause haemolysis in fetus this can also occur in the first pregnancy due to leaks

Prevention

test for D antibodies in all Rh -ve mothers at booking NICE (2008) advise giving anti-D to Rh -ve mothers at 28 and 34 weeks the evidence base suggests that there is little difference in the efficacy of single-dose (at 28 weeks) and double-dose regimes (at 28 & 34 weeks). For this reason the RCOG in 2011 advised that either regime could be used 'depending on local factors' anti-D is prophylaxis - once sensitization has occurred it is irreversible if event is in 2nd/3rd trimester give large dose of anti-D and perform Kleihauer test - determines proportion of fetal RBCs present

Anti-D immunoglobulin should be given as soon as possible (but always within 72 hours) in the following situations:

delivery of a Rh +ve infant, whether live or stillborn any termination of pregnancy miscarriage if gestation is > 12 weeks ectopic pregnancy

external cephalic version antepartum haemorrhage amniocentesis, chorionic villus sampling, fetal blood sampling

Tests

all babies born to Rh -ve mother should have cord blood taken at delivery for FBC, blood group & direct Coombs test Coombs test: direct antiglobulin , will demonstrate antibodies on RBCs of baby Kleihauer test: add acid to maternal blood, fetal cells are resistant

Affected fetus

oedematous (hydrops fetalis, as liver devoted to RBC production albumin falls) jaundice, anaemia, hepatosplenomegaly heart failure kernicterus treatment: transfusions, UV phototherapy dr.naila@yahoo.com - Applied Knowledge Test - My account

44Go

Search

External links Australian goverment Prescribing in pregnancy

A 37-year-old woman with a history of type 2 diabetes mellitus and obesity presents after a late period. The urinary hCG test is positive. Her current medication is as follows: Orlistat 120mg tds Simvastatin 40mg on Aspirin 75mg od Metformin 1g bd Paracetamol 1g qds Aqueous cream prn Which one of her medications must be stopped straight away? A. Paracetamol B. Aspirin C. Simvastatin D. Orlistat E. Metformin Simvastatin is contraindicated in pregnancy and must be stopped immediately. Metformin is sometimes used in pregnancy although many diabetic women are converted to insulin for the duration of the pregnancy to try and maximise control and minimise complications. Whilst orlistat is not a known teratogen it should be used with 'caution' in pregnancy according to the BNF and the benefits are very likely outweighed by risks.

Prescribing in pregnant patients Very few drugs are known to be completely safe in pregnancy. The list below largely comprises of those known to be harmful. Some countries have developed a grading system - see the link. Antibiotics

tetracyclines aminoglycosides sulphonamides and trimethoprim quinolones: the BNF advises to avoid due to arthropathy in some animal studies

Other drugs

ACE inhibitors, angiotensin II receptor antagonists statins warfarin sulfonylureas retinoids (including topical) cytotoxic agents

The majority of antiepileptics including valproate, carbamazepine and phenytoin are known to be potentially harmful. The decision to stop such treatments however is difficult as uncontrolled epilepsy is also a risk dr.naila@yahoo.com - Applied Knowledge Test - My account 45RCGP curriculum

3.06 - Womens Health Search

Go External links NICE 2008 Management of diabetes in pregnancy SIGN Management of diabetes in pregnancy Postgraduate Medical Journal DKA in pregnancy Which one of the following statements regarding the management of diabetes mellitus during pregnancy is incorrect? A. A previous macrosomic baby is a risk factor for gestational diabetes B. Diabetes complicates around 1 in 40 pregnancies C. A higher dose of folic acid (5 mg/day) should be used D. Metformin is contraindicated E. Tight glycaemic control reduces complication rates There is increasing evidence that metformin is safe during pregnancy Pregnancy: diabetes mellitus

Diabetes mellitus may be a pre-existing problem or develop during pregnancy, gestational diabetes. It complicates around 1 in 40 pregnancies Risk factors for gestational diabetes

BMI of > 30 kg/m^2 previous macrosomic baby weighing 4.5 kg or above. previous gestational diabetes first-degree relative with diabetes family origin with a high prevalence of diabetes (South Asian, black Caribbean and Middle Eastern)

Screening for gestational diabetes

if a women has had gestational diabetes previously an oral glucose tolerance test (OGTT) should be performed at 16-18 weeks and at 28 weeks if the first test is normal women with any of the other risk factors should be offered an OGTT at 24-28 weeks currently the same WHO diagnostic criteria are used as for non-pregnant patients. There is however increasing evidence that a lower threshold should be used as treating borderline patients improves both maternal and neonatal outcomes

NICE issued guidelines on the management of diabetes mellitus in pregnancy in 2008 Management of pre-existing diabetes

weight loss for women with BMI of > 27 kg/m^2

stop oral hypoglycaemic agents, apart from metformin, and commence insulin folic acid 5 mg/day from pre-conception to 12 weeks gestation detailed anomaly scan at 18-20 weeks including fourchamber view of the heart and outflow tracts tight glycaemic control reduces complication rates treat retinopathy as can worsen during pregnancy

Management of gestational diabetes


responds to changes in diet and exercise in around 80% of women oral hypoglycaemic agents (metformin or glibenclamide) or insulin injections are needed if blood glucose control is poor or this is any evidence of complications (e.g. macrosomia) there is increasing evidence that oral hypoglycaemic agents are both safe and give similar outcomes to insulin hypoglycaemic medication should be stopped following delivery a fasting glucose should be checked at the 6 week postnatal check dr.naila@yahoo.com - Applied Knowledge Test - My account

46-

RCGP curriculum

3.06 - Womens Health Search Go Which one of the following statements regarding preterm birth is incorrect?

A. Is defined as delivery of an infant before 37 weeks gestation B. Around 45% of twin pregnancies are premature C. Occurs in around 15-20% of pregnancies D. Diabetes mellitus is a risk factor E. May be elective due to congenital abnormalities

Preterm birth occurs in around 5-10% of pregnancies Preterm birth Preterm birth is defined as delivery of an infant before 37 weeks gestation. It occurs in around 5-10% of pregnancies (6% of singletons, 45% of twins) Causes
123456789-

Unexplained (30-40%) Multiple pregnancies (20-30%) Congenital abnormalities Antepartum haemorrhage Pre-eclampsia Cervical incompetence Diabetes mellitus Polyhydramnios Uterine abnormalities

10-

Infections e.g. Pyelonephritis dr.naila@yahoo.com - Applied Knowledge Test - My account

47-

RCGP curriculum

3.06 - Womens Health Search Go External links RCOG 2006 Management of severe pre-eclampsia/eclampsia NICE 2010 Hypertension in pregnancy A 29-year-old woman who is 28 weeks pregnant is reviewed. She has developed pre-eclampsia with her current blood pressure being 156/104 mmHg and the urine dipstick reported as follows: Protein +

Leucocytes negative Blood negative

There is no oedema and the patient is otherwise asymptomatic. Of the following drugs, which one is least suitable to use? A. Labetalol

B. Nifedipine C. Losartan D. Methyldopa E. Hydralazine ACE inhibitors and angiotensin-2 receptor blockers should be avoided as they are teratogenic. Most clinicians would either use methyldopa or labetalol first-line in this situation Pre-eclampsia Pre-eclampsia is a condition seen after 20 weeks gestation characterised by pregnancy-induced hypertension in association with proteinuria (> 0.3g / 24 hours). Oedema used to be third element of the classic triad but is now often not included in the definition as it is not specific Pre-eclampsia is important as it predisposes to the following problems

fetal: prematurity, intrauterine growth retardation eclampsia haemorrhage: placental abruption, intra-abdominal, intracerebral cardiac failure multi-organ failure

Risk factors

> 40 years old nulliparity (or new partner) multiple pregnancy body mass index > 30 kg/m^2 diabetes mellitus pregnancy interval of more than 10 years family history of pre-eclampsia previous history of pre-eclampsia pre-existing vascular disease such as hypertension or renal disease

Features of severe pre-eclampsia


hypertension: typically > 170/110 mmHg and proteinuria as above proteinuria: dipstick ++/+++ headache visual disturbance papilloedema RUQ/epigastric pain hyperreflexia platelet count < 100 * 106/l, abnormal liver enzymes or HELLP syndrome

Management

consensus guidelines recommend treating blood pressure > 160/110 mmHg although many clinicians have a lower threshold oral labetalol is now first-line following the 2010 NICE guidelines. Nifedipine and hydralazine may also be used delivery of the baby is the most important and definitive management step. The timing depends on the individual clinical scenario

dr.naila@yahoo.com - Applied Knowledge Test - My account 48RCGP curriculum

3.06 - Womens Health Search Go External links RCOG Post-partum haemorrhage guidelines A 29-year-old woman loses around 1,000 ml of blood shortly following a vaginal delivery. Which one of the following is not a risk factor for primary post-partum haemorrhage? A. Macrosomia B. Pre-eclampsia C. Afro-Caribbean ethnicity D. Prolonged labour E. Polyhydramnios

Post-partum haemorrhage

Post-partum haemorrhage (PPH) is defined as blood loss of > 500mls and may be primary or secondary Primary PPH

occurs within 24 hours affects around 5-7% of deliveries most common cause of PPH is uterine atony (90% of cases). Other causes include genital trauma and clotting factors

Risk factors for primary PPH include*:


previous PPH prolonged labour pre-eclampsia increased maternal age polyhydramnios emergency Caesarean section placenta praevia macrosomia ritodrine (a beta-2 adrenergic receptor agonist used for tocolysis)

Management

ABC IV syntocinon (oxytocin) 10 units or IV ergometrine 500 mcg IM carboprost other options include: B-Lynch suture, ligation of the uterine arteries or internal iliac arteries if severe, uncontrolled haemorrhage then a hysterectomy is sometimes performed as a life-saving procedure

Secondary PPH

occurs between 24 hours - 12 weeks** due to retained placental tissue or endometritis

*the effect of parity on the risk of PPH is complicated. It was previously though multiparity was a risk factor but more modern studies suggest nulliparity is actually a risk factor **previously the definition of secondary PPH was 24 hours - 6 weeks. Please see the RCOG guidelines for more details dr.naila@yahoo.com - Applied Knowledge Test - My account

49-

RCGP curriculum

3.06 - Womens Health Search Go Which one of the following statements regarding amniocentesis is incorrect? A. Is performed under ultrasound guidance B. Is usually performed at 16 weeks C. Can help detect neural tube defects

D. Karyotype may be wrong in 1 in 1,000 cases E. Risk of fetal loss is 2-3%

Amniocentesis: risk of fetal loss is 0.5-1% Amniocentesis Amniocentesis is a procedure used in prenatal diagnosis. It may be offered after screening tests have indicated a high risk of fetal abnormality or in women considered to be at high risk, for example if > 35 years old. Around 20 ml of fluid is removed by trans-abdominal needle under ultrasound guidance. Fetal cells present in the amniotic fluid are then studied to aid the diagnosis of a number of conditions. Amniocentesis is usually performed at 16 weeks and the risk of fetal loss is 0.5-1%. The karyotype results typically take 3 weeks. It is known the karyotype may be wrong in 1/1000 cases due to maternal cells being present Conditions which may be diagnosed

Neural tube defects (raised AFP levels in the amniotic fluid) Chromosomal disorders Inborn errors of metabolism dr.naila@yahoo.com - Applied Knowledge Test - My account

50-

RCGP curriculum

3.06 - Womens Health Search Go Which one of the following regarding the management of thyroid problems during pregnancy is incorrect? A. Maternal free thyroxine levels should be kept in the upper third of the normal reference range when treating thyrotoxicosis B. Increased levels of thyroxine-binding globulin are seen in pregnancy C. Block-and-replace is preferable in pregnancy compared to antithyroid drug titration D. Breast feeding is safe whilst on thyroxine E. Untreated thyrotoxicosis increases the risk of premature labour Pregnancy: thyroid problems In pregnancy there is an increase in the levels of thyroxinebinding globulin (TBG). This causes an increase in the levels of total thyroxine but does not affect the free thyroxine level Thyrotoxicosis

Un-treated thyrotoxicosis increases the risk of fetal loss, maternal heart failure and premature labor Graves' disease is the most common cause of thyrotoxicosis in pregnancy. It is also recognized that activation of the TSH receptor by HCG may also occur - often termed transient gestational hyperthyroidism. HCG levels will fall in second and third trimester Management

Propylthiouracil has traditionally been the antithyroid drug of choice. This approach was supported by the 2007 Endocrine Society consensus guidelines Maternal free thyroxine levels should be kept in the upper third of the normal reference range to avoid fetal hypothyroidism Thyrotrophin receptor stimulating antibodies should be checked at 30-36 weeks gestation - helps to determine risk of neonatal thyroid problems Block-and-replace regimes should not be used in pregnancy Radioiodine therapy is contraindicated

Hypothyroidism Key points


Thyroxine is safe during pregnancy Serum thyroid stimulating hormone measured in each trimester and 6-8 weeks post-partum Some women require an increased dose of thyroxine during pregnancy Breast feeding is safe whilst on thyroxine dr.naila@yahoo.com - Applied Knowledge Test - My account

51-

RCGP curriculum

3.06 - Womens Health Search Go A pregnant woman presents for review. She is 24 weeks pregnant. What would be the expected symphysis-fundal height? A. 13 - 15 cm B. 15 - 17 cm C. 17 - 19 cm D. 18 - 22 cm E. 22 - 26 cm

After 20 weeks, symphysis-fundal height in cm = gestation in weeks

Symphysis-fundal height The symphysis-fundal height (SFH) is measured from the top of the pubic bone to the top of the uterus in centimetres

It should match the gestational age in weeks to within 2 cm after 20 weeks, e.g. if 24 weeks then the a normal SFH = 22 to 26 cm dr.naila@yahoo.com - Applied Knowledge Test - My account 52RCGP curriculum

3.06 - Womens Health Search Go External links RCOG 2006 Management of severe pre-eclampsia/eclampsia NICE 2010 Hypertension in pregnancy Which one of the following is not a risk factor for the development of pre-eclampsia? A. Previous history of pre-eclampsia B. Body mass index of 34 kg/m^2 C. Age of 42 years D. Multiple pregnancy E. Multiparity

No previous pregnancies is a risk factor for pre-eclampsia Pre-eclampsia Pre-eclampsia is a condition seen after 20 weeks gestation characterised by pregnancy-induced hypertension in association with proteinuria (> 0.3g / 24 hours). Oedema used to be third element of the classic triad but is now often not included in the definition as it is not specific Pre-eclampsia is important as it predisposes to the following problems

fetal: prematurity, intrauterine growth retardation eclampsia haemorrhage: placental abruption, intra-abdominal, intracerebral cardiac failure multi-organ failure

Risk factors

> 40 years old nulliparity (or new partner) multiple pregnancy body mass index > 30 kg/m^2 diabetes mellitus pregnancy interval of more than 10 years family history of pre-eclampsia previous history of pre-eclampsia pre-existing vascular disease such as hypertension or renal disease

Features of severe pre-eclampsia


hypertension: typically > 170/110 mmHg and proteinuria as above proteinuria: dipstick ++/+++ headache visual disturbance papilloedema RUQ/epigastric pain hyperreflexia platelet count < 100 * 106/l, abnormal liver enzymes or HELLP syndrome

Management

consensus guidelines recommend treating blood pressure > 160/110 mmHg although many clinicians have a lower threshold oral labetalol is now first-line following the 2010 NICE guidelines. Nifedipine and hydralazine may also be used delivery of the baby is the most important and definitive management step. The timing depends on the individual clinical scenario dr.naila@yahoo.com - Applied Knowledge Test - My account

53-

RCGP curriculum

3.06 - Womens Health Search Go

External links RCOG 2006 Management of severe pre-eclampsia/eclampsia NICE 2010 Hypertension in pregnancy Which one of the following is not a risk factor for the development of pre-eclampsia? A. Body mass index of 38 kg/m^2 B. Smoking C. A woman carrying twins D. Nulliparity E. Diabetes mellitus There is some evidence to suggest that pre-eclampsia is actually less common in smokers Pre-eclampsia Pre-eclampsia is a condition seen after 20 weeks gestation characterised by pregnancy-induced hypertension in association with proteinuria (> 0.3g / 24 hours). Oedema used to be third element of the classic triad but is now often not included in the definition as it is not specific Pre-eclampsia is important as it predisposes to the following problems

fetal: prematurity, intrauterine growth retardation eclampsia haemorrhage: placental abruption, intra-abdominal, intracerebral cardiac failure multi-organ failure

Risk factors

> 40 years old nulliparity (or new partner) multiple pregnancy body mass index > 30 kg/m^2 diabetes mellitus pregnancy interval of more than 10 years family history of pre-eclampsia previous history of pre-eclampsia pre-existing vascular disease such as hypertension or renal disease

Features of severe pre-eclampsia


hypertension: typically > 170/110 mmHg and proteinuria as above proteinuria: dipstick ++/+++ headache visual disturbance papilloedema RUQ/epigastric pain hyperreflexia platelet count < 100 * 106/l, abnormal liver enzymes or HELLP syndrome

Management

consensus guidelines recommend treating blood pressure > 160/110 mmHg although many clinicians have a lower threshold oral labetalol is now first-line following the 2010 NICE guidelines. Nifedipine and hydralazine may also be used delivery of the baby is the most important and definitive management step. The timing depends on the individual clinical scenario dr.naila@yahoo.com - Applied Knowledge Test - My account

54-

RCGP curriculum

3.06 - Womens Health Search Go A woman who is 41 weeks pregnant has a biophysical profile. Which one of the following is not assessed during this test? A. Fetal activity B. Head circumference C. Fetal breathing movements D. Fetal tone E. Amniotic fluid volume

Biophysical profile A biophysical profile is an antenatal ultrasound test which assesses:


12345-

Amniotic fluid volume Fetal tone Fetal activity Fetal breathing movements Reactivity of the heart

55-

External links

NICE 2008 Routine anti-D prophylaxis guidelines RCOG Use of Anti-D Immunoglobulin for Rh Prophylaxis You are reviewing the blood results for a pregnant woman. Which one of the following results would indicate the need for routine antenatal anti-D prophylaxis to be given at 28 weeks? A. Rhesus negative mothers who are not sensitised B. Rhesus negative mothers who are sensitised C. Rhesus negative mothers who are sensitised where there is coexistent anaemia

D. Rhesus positive mothers who are sensitised E. Rhesus positive mothers who are not sensitised Rhesus negative pregnancy A basic understanding of the pathophysiology is essential to understand the management of Rhesus negative pregnancies

along with the ABO system the Rhesus system is the most important antigen found on red blood cells. The D antigen is the most important antigen of the rhesus system around 15% of mothers are rhesus negative (Rh -ve) if a Rh -ve mother delivers a Rh +ve child a leak of fetal red blood cells may occur this causes anti-D IgG antibodies to form in mother in later pregnancies these can cross placenta and cause haemolysis in fetus this can also occur in the first pregnancy due to leaks

Prevention

test for D antibodies in all Rh -ve mothers at booking NICE (2008) advise giving anti-D to non-sensitised Rh -ve mothers at 28 and 34 weeks the evidence base suggests that there is little difference in the efficacy of single-dose (at 28 weeks) and double-dose regimes (at 28 & 34 weeks). For this reason the RCOG in 2011 advised that either regime could be used 'depending on local factors' anti-D is prophylaxis - once sensitization has occurred it is irreversible

if event is in 2nd/3rd trimester give large dose of anti-D and perform Kleihauer test - determines proportion of fetal RBCs present

Anti-D immunoglobulin should be given as soon as possible (but always within 72 hours) in the following situations:

delivery of a Rh +ve infant, whether live or stillborn any termination of pregnancy miscarriage if gestation is > 12 weeks ectopic pregnancy external cephalic version antepartum haemorrhage amniocentesis, chorionic villus sampling, fetal blood sampling

Tests

all babies born to Rh -ve mother should have cord blood taken at delivery for FBC, blood group & direct Coombs test Coombs test: direct antiglobulin, will demonstrate antibodies on RBCs of baby Kleihauer test: add acid to maternal blood, fetal cells are resistant

Affected fetus

oedematous (hydrops fetalis, as liver devoted to RBC production albumin falls) jaundice, anaemia, hepatosplenomegaly heart failure kernicterus treatment: transfusions, UV phototherapy

You might also like